SafeCracker #3 - We've Been Blocked [on hold]What are the features of a well-written puzzle?I've been everywhere, manCircuit DiagramA riddle that has been killing me the whole daySafeCracker #1 - Look who's talkingSafeCracker #2 - The Mission Continues

How to preserve a rare version of a book?

HSA - Continue to Invest?

Huffman Code in C++

What is the meaning of 「隣のおじいさんは言いました」

What detail can Hubble see on Mars?

What do you call a painting painted on a wall?

Game artist computer workstation set-up – is this overkill?

Which "exotic salt" can lower water's freezing point by –70 °C?

Changing stroke width vertically but not horizontally in Inkscape

How to replace space with '+' symbol in a triangular array?

Can anyone identify this unknown 1988 PC card from The Palantir Corporation?

Efficient deletion of specific list entries

How did the Apollo guidance computer handle parity bit errors?

Can I combine SELECT TOP() with the IN operator?

How is trade in services conducted under the WTO in the absence of the Doha conclusion?

How to deal with employer who keeps me at work after working hours

Can a good but unremarkable PhD student become an accomplished professor?

As a GM, is it bad form to ask for a moment to think when improvising?

Is crescere the correct word meaning to to grow or cultivate?

Emergency stop in plain TeX, pdfTeX, XeTeX and LuaTeX?

Ab major 9th chord in Bach

Installing Debian 10, upgrade to stable later?

What does the copyright in a dissertation protect exactly?

Can an earth elemental drag a tiny creature underground with Earth Glide?



SafeCracker #3 - We've Been Blocked [on hold]


What are the features of a well-written puzzle?I've been everywhere, manCircuit DiagramA riddle that has been killing me the whole daySafeCracker #1 - Look who's talkingSafeCracker #2 - The Mission Continues













-1












$begingroup$


SafeCracker 3enter image description here



Our Last Safe To Conquer



Ok, team, this one might give us a little trouble. Nothing we can't handle though... let's go...



This safe model is "The NevaCrak 9000" - the most secure safe there is.



Our only way in is to look at our puzzle, and simply find the block that doesn't belong.



A couple blocks are blank and are to be ignored. Only concern yourself with blocks with numbers and letters. Also disregard any of the flaire going on in the background.



THAT's IT



Everything is riding on you... go open that bad boy!




And here is the NevaCrak 9000




Which block does not belong??



( only one correct answer, no tricks or worplay involved )



The Safe










share|improve this question











$endgroup$



put on hold as too broad by noedne, PiIsNot3, Brandon_J, Peregrine Rook, Omega Krypton 2 days ago


Please edit the question to limit it to a specific problem with enough detail to identify an adequate answer. Avoid asking multiple distinct questions at once. See the How to Ask page for help clarifying this question. If this question can be reworded to fit the rules in the help center, please edit the question.













  • 5




    $begingroup$
    @JohnS. "What comes next number puzzles" can also have many correct answers...
    $endgroup$
    – noedne
    May 2 at 17:25






  • 3




    $begingroup$
    @JohnS. Also, I’m not sure that insulting the puzzle solver’s intelligence will suddenly make them figure out the answer...
    $endgroup$
    – PiIsNot3
    May 2 at 22:57






  • 5




    $begingroup$
    @JohnS. This has nothing to do with being a new member but rather the content of the puzzle itself. We’re all trying to provide guidance on how you can improve your questions and what’s considered on-topic here. I sincerely hope you don’t take any of our criticism personally! :)
    $endgroup$
    – PiIsNot3
    May 2 at 23:32






  • 6




    $begingroup$
    @JohnS. I understand your frustration. You wrote a puzzle with a solution that clearly works when you find it. But I encourage you to think for a moment from the puzzle solver's perspective. Ak19 came up with a rule that works perfectly well to find an odd block out. From their perspective, they have no way of knowing that this is not the solution you intended. When you tell them it's not what you had in mind, it may feel unfair that their valid solution is deemed incorrect. It's important for puzzles to have unique solutions, but it's easy to miss things no matter how good your intentions.
    $endgroup$
    – noedne
    May 3 at 3:13






  • 4




    $begingroup$
    @JohnS. Compare the current question to a similar one: "What is the odd one out among the values 1, 2, 3, 4, 5?" Without any additional specifications from the author, any number of answers would be equally valid (5, because it's the only multiple of 5; 4, because it's the only composite number; etc. etc.), and thus the "correct" answer becomes a subjective choice. It's not the task itself that is problematic, but rather determining whether a possible answer is right or not. Also, I'm not sure how being a paid member of SE is relevant to this conversation.
    $endgroup$
    – PiIsNot3
    2 days ago
















-1












$begingroup$


SafeCracker 3enter image description here



Our Last Safe To Conquer



Ok, team, this one might give us a little trouble. Nothing we can't handle though... let's go...



This safe model is "The NevaCrak 9000" - the most secure safe there is.



Our only way in is to look at our puzzle, and simply find the block that doesn't belong.



A couple blocks are blank and are to be ignored. Only concern yourself with blocks with numbers and letters. Also disregard any of the flaire going on in the background.



THAT's IT



Everything is riding on you... go open that bad boy!




And here is the NevaCrak 9000




Which block does not belong??



( only one correct answer, no tricks or worplay involved )



The Safe










share|improve this question











$endgroup$



put on hold as too broad by noedne, PiIsNot3, Brandon_J, Peregrine Rook, Omega Krypton 2 days ago


Please edit the question to limit it to a specific problem with enough detail to identify an adequate answer. Avoid asking multiple distinct questions at once. See the How to Ask page for help clarifying this question. If this question can be reworded to fit the rules in the help center, please edit the question.













  • 5




    $begingroup$
    @JohnS. "What comes next number puzzles" can also have many correct answers...
    $endgroup$
    – noedne
    May 2 at 17:25






  • 3




    $begingroup$
    @JohnS. Also, I’m not sure that insulting the puzzle solver’s intelligence will suddenly make them figure out the answer...
    $endgroup$
    – PiIsNot3
    May 2 at 22:57






  • 5




    $begingroup$
    @JohnS. This has nothing to do with being a new member but rather the content of the puzzle itself. We’re all trying to provide guidance on how you can improve your questions and what’s considered on-topic here. I sincerely hope you don’t take any of our criticism personally! :)
    $endgroup$
    – PiIsNot3
    May 2 at 23:32






  • 6




    $begingroup$
    @JohnS. I understand your frustration. You wrote a puzzle with a solution that clearly works when you find it. But I encourage you to think for a moment from the puzzle solver's perspective. Ak19 came up with a rule that works perfectly well to find an odd block out. From their perspective, they have no way of knowing that this is not the solution you intended. When you tell them it's not what you had in mind, it may feel unfair that their valid solution is deemed incorrect. It's important for puzzles to have unique solutions, but it's easy to miss things no matter how good your intentions.
    $endgroup$
    – noedne
    May 3 at 3:13






  • 4




    $begingroup$
    @JohnS. Compare the current question to a similar one: "What is the odd one out among the values 1, 2, 3, 4, 5?" Without any additional specifications from the author, any number of answers would be equally valid (5, because it's the only multiple of 5; 4, because it's the only composite number; etc. etc.), and thus the "correct" answer becomes a subjective choice. It's not the task itself that is problematic, but rather determining whether a possible answer is right or not. Also, I'm not sure how being a paid member of SE is relevant to this conversation.
    $endgroup$
    – PiIsNot3
    2 days ago














-1












-1








-1


1



$begingroup$


SafeCracker 3enter image description here



Our Last Safe To Conquer



Ok, team, this one might give us a little trouble. Nothing we can't handle though... let's go...



This safe model is "The NevaCrak 9000" - the most secure safe there is.



Our only way in is to look at our puzzle, and simply find the block that doesn't belong.



A couple blocks are blank and are to be ignored. Only concern yourself with blocks with numbers and letters. Also disregard any of the flaire going on in the background.



THAT's IT



Everything is riding on you... go open that bad boy!




And here is the NevaCrak 9000




Which block does not belong??



( only one correct answer, no tricks or worplay involved )



The Safe










share|improve this question











$endgroup$




SafeCracker 3enter image description here



Our Last Safe To Conquer



Ok, team, this one might give us a little trouble. Nothing we can't handle though... let's go...



This safe model is "The NevaCrak 9000" - the most secure safe there is.



Our only way in is to look at our puzzle, and simply find the block that doesn't belong.



A couple blocks are blank and are to be ignored. Only concern yourself with blocks with numbers and letters. Also disregard any of the flaire going on in the background.



THAT's IT



Everything is riding on you... go open that bad boy!




And here is the NevaCrak 9000




Which block does not belong??



( only one correct answer, no tricks or worplay involved )



The Safe







number-sequence formation-of-numbers grid-deduction algorithm algebra






share|improve this question















share|improve this question













share|improve this question




share|improve this question








edited May 2 at 17:51







John S.

















asked May 2 at 15:29









John S.John S.

52412




52412




put on hold as too broad by noedne, PiIsNot3, Brandon_J, Peregrine Rook, Omega Krypton 2 days ago


Please edit the question to limit it to a specific problem with enough detail to identify an adequate answer. Avoid asking multiple distinct questions at once. See the How to Ask page for help clarifying this question. If this question can be reworded to fit the rules in the help center, please edit the question.









put on hold as too broad by noedne, PiIsNot3, Brandon_J, Peregrine Rook, Omega Krypton 2 days ago


Please edit the question to limit it to a specific problem with enough detail to identify an adequate answer. Avoid asking multiple distinct questions at once. See the How to Ask page for help clarifying this question. If this question can be reworded to fit the rules in the help center, please edit the question.









  • 5




    $begingroup$
    @JohnS. "What comes next number puzzles" can also have many correct answers...
    $endgroup$
    – noedne
    May 2 at 17:25






  • 3




    $begingroup$
    @JohnS. Also, I’m not sure that insulting the puzzle solver’s intelligence will suddenly make them figure out the answer...
    $endgroup$
    – PiIsNot3
    May 2 at 22:57






  • 5




    $begingroup$
    @JohnS. This has nothing to do with being a new member but rather the content of the puzzle itself. We’re all trying to provide guidance on how you can improve your questions and what’s considered on-topic here. I sincerely hope you don’t take any of our criticism personally! :)
    $endgroup$
    – PiIsNot3
    May 2 at 23:32






  • 6




    $begingroup$
    @JohnS. I understand your frustration. You wrote a puzzle with a solution that clearly works when you find it. But I encourage you to think for a moment from the puzzle solver's perspective. Ak19 came up with a rule that works perfectly well to find an odd block out. From their perspective, they have no way of knowing that this is not the solution you intended. When you tell them it's not what you had in mind, it may feel unfair that their valid solution is deemed incorrect. It's important for puzzles to have unique solutions, but it's easy to miss things no matter how good your intentions.
    $endgroup$
    – noedne
    May 3 at 3:13






  • 4




    $begingroup$
    @JohnS. Compare the current question to a similar one: "What is the odd one out among the values 1, 2, 3, 4, 5?" Without any additional specifications from the author, any number of answers would be equally valid (5, because it's the only multiple of 5; 4, because it's the only composite number; etc. etc.), and thus the "correct" answer becomes a subjective choice. It's not the task itself that is problematic, but rather determining whether a possible answer is right or not. Also, I'm not sure how being a paid member of SE is relevant to this conversation.
    $endgroup$
    – PiIsNot3
    2 days ago













  • 5




    $begingroup$
    @JohnS. "What comes next number puzzles" can also have many correct answers...
    $endgroup$
    – noedne
    May 2 at 17:25






  • 3




    $begingroup$
    @JohnS. Also, I’m not sure that insulting the puzzle solver’s intelligence will suddenly make them figure out the answer...
    $endgroup$
    – PiIsNot3
    May 2 at 22:57






  • 5




    $begingroup$
    @JohnS. This has nothing to do with being a new member but rather the content of the puzzle itself. We’re all trying to provide guidance on how you can improve your questions and what’s considered on-topic here. I sincerely hope you don’t take any of our criticism personally! :)
    $endgroup$
    – PiIsNot3
    May 2 at 23:32






  • 6




    $begingroup$
    @JohnS. I understand your frustration. You wrote a puzzle with a solution that clearly works when you find it. But I encourage you to think for a moment from the puzzle solver's perspective. Ak19 came up with a rule that works perfectly well to find an odd block out. From their perspective, they have no way of knowing that this is not the solution you intended. When you tell them it's not what you had in mind, it may feel unfair that their valid solution is deemed incorrect. It's important for puzzles to have unique solutions, but it's easy to miss things no matter how good your intentions.
    $endgroup$
    – noedne
    May 3 at 3:13






  • 4




    $begingroup$
    @JohnS. Compare the current question to a similar one: "What is the odd one out among the values 1, 2, 3, 4, 5?" Without any additional specifications from the author, any number of answers would be equally valid (5, because it's the only multiple of 5; 4, because it's the only composite number; etc. etc.), and thus the "correct" answer becomes a subjective choice. It's not the task itself that is problematic, but rather determining whether a possible answer is right or not. Also, I'm not sure how being a paid member of SE is relevant to this conversation.
    $endgroup$
    – PiIsNot3
    2 days ago








5




5




$begingroup$
@JohnS. "What comes next number puzzles" can also have many correct answers...
$endgroup$
– noedne
May 2 at 17:25




$begingroup$
@JohnS. "What comes next number puzzles" can also have many correct answers...
$endgroup$
– noedne
May 2 at 17:25




3




3




$begingroup$
@JohnS. Also, I’m not sure that insulting the puzzle solver’s intelligence will suddenly make them figure out the answer...
$endgroup$
– PiIsNot3
May 2 at 22:57




$begingroup$
@JohnS. Also, I’m not sure that insulting the puzzle solver’s intelligence will suddenly make them figure out the answer...
$endgroup$
– PiIsNot3
May 2 at 22:57




5




5




$begingroup$
@JohnS. This has nothing to do with being a new member but rather the content of the puzzle itself. We’re all trying to provide guidance on how you can improve your questions and what’s considered on-topic here. I sincerely hope you don’t take any of our criticism personally! :)
$endgroup$
– PiIsNot3
May 2 at 23:32




$begingroup$
@JohnS. This has nothing to do with being a new member but rather the content of the puzzle itself. We’re all trying to provide guidance on how you can improve your questions and what’s considered on-topic here. I sincerely hope you don’t take any of our criticism personally! :)
$endgroup$
– PiIsNot3
May 2 at 23:32




6




6




$begingroup$
@JohnS. I understand your frustration. You wrote a puzzle with a solution that clearly works when you find it. But I encourage you to think for a moment from the puzzle solver's perspective. Ak19 came up with a rule that works perfectly well to find an odd block out. From their perspective, they have no way of knowing that this is not the solution you intended. When you tell them it's not what you had in mind, it may feel unfair that their valid solution is deemed incorrect. It's important for puzzles to have unique solutions, but it's easy to miss things no matter how good your intentions.
$endgroup$
– noedne
May 3 at 3:13




$begingroup$
@JohnS. I understand your frustration. You wrote a puzzle with a solution that clearly works when you find it. But I encourage you to think for a moment from the puzzle solver's perspective. Ak19 came up with a rule that works perfectly well to find an odd block out. From their perspective, they have no way of knowing that this is not the solution you intended. When you tell them it's not what you had in mind, it may feel unfair that their valid solution is deemed incorrect. It's important for puzzles to have unique solutions, but it's easy to miss things no matter how good your intentions.
$endgroup$
– noedne
May 3 at 3:13




4




4




$begingroup$
@JohnS. Compare the current question to a similar one: "What is the odd one out among the values 1, 2, 3, 4, 5?" Without any additional specifications from the author, any number of answers would be equally valid (5, because it's the only multiple of 5; 4, because it's the only composite number; etc. etc.), and thus the "correct" answer becomes a subjective choice. It's not the task itself that is problematic, but rather determining whether a possible answer is right or not. Also, I'm not sure how being a paid member of SE is relevant to this conversation.
$endgroup$
– PiIsNot3
2 days ago





$begingroup$
@JohnS. Compare the current question to a similar one: "What is the odd one out among the values 1, 2, 3, 4, 5?" Without any additional specifications from the author, any number of answers would be equally valid (5, because it's the only multiple of 5; 4, because it's the only composite number; etc. etc.), and thus the "correct" answer becomes a subjective choice. It's not the task itself that is problematic, but rather determining whether a possible answer is right or not. Also, I'm not sure how being a paid member of SE is relevant to this conversation.
$endgroup$
– PiIsNot3
2 days ago











10 Answers
10






active

oldest

votes


















7












$begingroup$

The odd one out is the




green block with 5246.




This is because




for all the other numbers on the other blocks, combining the first and last digits and dividing by 2 gave the middle two digits. However, 5246 does not apply to that rule. To conform it to that rule, it would have to be 4248 (one of many possible fixes)...




Safe Cracked!






share|improve this answer











$endgroup$












  • $begingroup$
    The answer should be a single block (the ones that have a letter and number); not a complete group of blocks
    $endgroup$
    – Daniel Duque
    May 2 at 17:52










  • $begingroup$
    Yes. Ok, I am giving a very small , yet critical Hint: It's all a numbers game, it's allll a numbers game.
    $endgroup$
    – John S.
    May 2 at 18:32






  • 1




    $begingroup$
    @JohnS. Didn't Ak19 correctly identify this block in an earlier answer?
    $endgroup$
    – noedne
    May 2 at 23:40






  • 2




    $begingroup$
    @JohnS. While Invent's original answer (divisible by 3) is an accurate pattern and a good find, Ak19 posted it first. It's also not the same as your "correct" answer. At all. You just got tired of waiting for someone to discover your solution and decided to give it away to someone.
    $endgroup$
    – David K
    2 days ago






  • 1




    $begingroup$
    No, there are many correct answers, as I have been pointed out. I wlll never make a puzzle with multiple solutions again. Not intentional anyways. He got a correct answer, we can't take that away from him. I am new here, so this is a learning lesson. I will be more specific in the future if I submit more. Thank you for your comment, though.
    $endgroup$
    – John S.
    2 days ago


















9












$begingroup$

The




green block with the number 5246




doesn't belong, because




it is the only number not divisible by 3 where all the others are.







share|improve this answer










New contributor




Ak19 is a new contributor to this site. Take care in asking for clarification, commenting, and answering.
Check out our Code of Conduct.






$endgroup$












  • $begingroup$
    Ak19, that is a very good suggested answer, however, it is not correct. You are thinking along the correct path, however.
    $endgroup$
    – John S.
    May 2 at 16:48










  • $begingroup$
    @Invent: 99.99999999%
    $endgroup$
    – John S.
    May 2 at 17:55






  • 1




    $begingroup$
    This is the right answer, for a slightly weaker reason than the ‘‘official’’ one.
    $endgroup$
    – Peregrine Rook
    May 3 at 2:20










  • $begingroup$
    all the numbers had exactly 3 prime factors. they were all (2,3,X) for mostly very large X (ranging from 11 to 1583). The only outlier is 5246 with prime factors of (2,43,61) once I saw this I couldn't look away!
    $endgroup$
    – stew
    2 days ago










  • $begingroup$
    @stew: Your comment is essentially the same as David K’s answer.
    $endgroup$
    – Peregrine Rook
    yesterday


















3












$begingroup$

I'm going to guess the:




Black block.

It's the only one that isn't a tetris piece :P







share|improve this answer









$endgroup$




















    3












    $begingroup$

    I will guess that it is




    Green block 5246




    My reasoning is that




    All other blocks have numbers that are divisible by 6, and it is the only block that has two prime factor other than 2 and 3. The prime factors of all other numbers are some number of 2s and 3s, and one other larger prime number. 5246 factorizes to 2, 43, and 61.







    share|improve this answer









    $endgroup$












    • $begingroup$
      This is equivalent to Ak19’s answer, but worded in a more complicated way.
      $endgroup$
      – Peregrine Rook
      May 3 at 2:21










    • $begingroup$
      @PeregrineRook It is an expansion of Ak19's answer, but they are not equivalent. The OP indicated that Ak19 was thinking along the correct path, so I dug deeper in that direction.
      $endgroup$
      – David K
      2 days ago






    • 1




      $begingroup$
      Ah, I see.  OK, you’re right.  Your answer encompasses Ak19’s answer, but you found something more. (Now it turns out that Ak19’s answer was very close to what the OP was looking for, and yours was not, but that doesn’t reflect badly on your answer.  It just goes to support the argument that the question is too broad, because it has too many different answers.) I would upvote you, but I already did.
      $endgroup$
      – Peregrine Rook
      yesterday


















    3












    $begingroup$

    My answer:




    When arranging the numbers in the order of the word Stack Exchange, and then doing an absolute value of the difference between any number and the following number, I get a pattern that the difference value rises then drops alternatingly, except at the Black A where the value remains the same. (Differences: 4176,8214,8214,3150,5088,1950,5274,2152,4252,0,4224,3126 - rise fall neutral fall rise fall rise fall rise fall rise fall)




    Below is Meant as comment, but alas



    @John S posted the following in a comment at https://puzzling.stackexchange.com/a/83526/60039




    [ C, 8412, E, 174,H, 5246,C, 7398,A, 9498,S, 4236,A, 198,T, 8412,N, 9498,G, 5274,E, 2148 ]




    It is missing data.




    The last three are the orange blocks, the second to last three are the black blocks, the third to last three are the green blocks and the first 2 are only half of the yellow blocks, the x and k are missing.




    However:




    There are two pairs letters with same numbers, 9498 and 8412.




    Also:




    John stated elsewhere its just a numbers game...which makes sense because the letters spell out Stack Exchange







    share|improve this answer










    New contributor




    wolfsshield is a new contributor to this site. Take care in asking for clarification, commenting, and answering.
    Check out our Code of Conduct.






    $endgroup$




















      2












      $begingroup$

      The answer is:




      The green block with the number 9498.




      because




      The average of the numbers on the green blocks is not a whole, even number (as it is for the other sets of blocks) and the number 9498 here is a duplicate of another block. This block could be 9496 which would make it valid.







      share|improve this answer










      New contributor




      Amru E. is a new contributor to this site. Take care in asking for clarification, commenting, and answering.
      Check out our Code of Conduct.






      $endgroup$




















        2












        $begingroup$

        Answer:




        The Black A




        Reason:




        It's the only one that has 11 as a factor







        share|improve this answer








        New contributor




        wolfsshield is a new contributor to this site. Take care in asking for clarification, commenting, and answering.
        Check out our Code of Conduct.






        $endgroup$




















          1












          $begingroup$

          I would guess it is the:




          Yellow block that has the E and the number 174




          Why?




          All other group of blocks follow the pattern of increasing/decreasing the number as the letters advance through the alphabet.







          share|improve this answer









          $endgroup$




















            0












            $begingroup$

            The answer might be:




            The yellow one




            Reasoning:




            All the other blocks have only letters when looked at from a specific direction.







            share|improve this answer









            $endgroup$












            • $begingroup$
              No, sorry. We are looking for an individual block that just doesn't fit the criteria of all the others. - But that was a reasonable guess!
              $endgroup$
              – John S.
              May 2 at 15:40











            • $begingroup$
              HERE IS THE DATA SET FOR THIS PUZZLE... Now, we are back to our regularly scheduled "What doesn't belong in this pattern?" puzzle. [ C, 8412, E, 174,H, 5246,C, 7398,A, 9498,S, 4236,A, 198,T, 8412,N, 9498,G, 5274,E, 2148 ]
              $endgroup$
              – John S.
              May 2 at 19:06











            • $begingroup$
              Delete this question. Not editing it. ANSWER WAS SIMPLE: take first number and last number , combine. Divide by 2. will give you middle number. ONLY ONE block doesn't fit criteria. People had fun trying, you can delete. Thanks.
              $endgroup$
              – John S.
              2 days ago


















            0












            $begingroup$

            I noticed that




            if you open out the net of each, centred on the side showing the letter, any numbers above, below and to the right are all aligned in the same direction as the letter; and the number on the left is on its side, with its top pointing to the left. The only exception is the Yellow K block, where the number on the right isn't aligned in the same direction as the letter.







            share|improve this answer








            New contributor




            MichaelMaggs is a new contributor to this site. Take care in asking for clarification, commenting, and answering.
            Check out our Code of Conduct.






            $endgroup$








            • 1




              $begingroup$
              Looks like there is more than one valid solution :(
              $endgroup$
              – MichaelMaggs
              May 2 at 16:51






            • 5




              $begingroup$
              If there's more than one block that's different from the others in a notable way and the challenge is to figure out what specific difference you had in mind, then the puzzle is basically "read the puzzle author's mind". Let's hope it turns out that the odd-one-out property John S intends is much more compelling than the two other perfectly plausible ones suggested so far...
              $endgroup$
              – Gareth McCaughan
              May 2 at 17:42






            • 1




              $begingroup$
              It is the other way around >! This is a hint.
              $endgroup$
              – Daniel Duque
              May 2 at 18:33






            • 1




              $begingroup$
              @JohnS. typically we use Rot13 to hide hints or spoilers in comments. But I’d strongly recommend you put the hints in the original post, since they would be considered part of the overall puzzle
              $endgroup$
              – PiIsNot3
              May 2 at 19:12







            • 1




              $begingroup$
              @JohnS. A "What's next?" like you describe should also be closed for being too broad...
              $endgroup$
              – noedne
              May 2 at 21:24

















            10 Answers
            10






            active

            oldest

            votes








            10 Answers
            10






            active

            oldest

            votes









            active

            oldest

            votes






            active

            oldest

            votes









            7












            $begingroup$

            The odd one out is the




            green block with 5246.




            This is because




            for all the other numbers on the other blocks, combining the first and last digits and dividing by 2 gave the middle two digits. However, 5246 does not apply to that rule. To conform it to that rule, it would have to be 4248 (one of many possible fixes)...




            Safe Cracked!






            share|improve this answer











            $endgroup$












            • $begingroup$
              The answer should be a single block (the ones that have a letter and number); not a complete group of blocks
              $endgroup$
              – Daniel Duque
              May 2 at 17:52










            • $begingroup$
              Yes. Ok, I am giving a very small , yet critical Hint: It's all a numbers game, it's allll a numbers game.
              $endgroup$
              – John S.
              May 2 at 18:32






            • 1




              $begingroup$
              @JohnS. Didn't Ak19 correctly identify this block in an earlier answer?
              $endgroup$
              – noedne
              May 2 at 23:40






            • 2




              $begingroup$
              @JohnS. While Invent's original answer (divisible by 3) is an accurate pattern and a good find, Ak19 posted it first. It's also not the same as your "correct" answer. At all. You just got tired of waiting for someone to discover your solution and decided to give it away to someone.
              $endgroup$
              – David K
              2 days ago






            • 1




              $begingroup$
              No, there are many correct answers, as I have been pointed out. I wlll never make a puzzle with multiple solutions again. Not intentional anyways. He got a correct answer, we can't take that away from him. I am new here, so this is a learning lesson. I will be more specific in the future if I submit more. Thank you for your comment, though.
              $endgroup$
              – John S.
              2 days ago















            7












            $begingroup$

            The odd one out is the




            green block with 5246.




            This is because




            for all the other numbers on the other blocks, combining the first and last digits and dividing by 2 gave the middle two digits. However, 5246 does not apply to that rule. To conform it to that rule, it would have to be 4248 (one of many possible fixes)...




            Safe Cracked!






            share|improve this answer











            $endgroup$












            • $begingroup$
              The answer should be a single block (the ones that have a letter and number); not a complete group of blocks
              $endgroup$
              – Daniel Duque
              May 2 at 17:52










            • $begingroup$
              Yes. Ok, I am giving a very small , yet critical Hint: It's all a numbers game, it's allll a numbers game.
              $endgroup$
              – John S.
              May 2 at 18:32






            • 1




              $begingroup$
              @JohnS. Didn't Ak19 correctly identify this block in an earlier answer?
              $endgroup$
              – noedne
              May 2 at 23:40






            • 2




              $begingroup$
              @JohnS. While Invent's original answer (divisible by 3) is an accurate pattern and a good find, Ak19 posted it first. It's also not the same as your "correct" answer. At all. You just got tired of waiting for someone to discover your solution and decided to give it away to someone.
              $endgroup$
              – David K
              2 days ago






            • 1




              $begingroup$
              No, there are many correct answers, as I have been pointed out. I wlll never make a puzzle with multiple solutions again. Not intentional anyways. He got a correct answer, we can't take that away from him. I am new here, so this is a learning lesson. I will be more specific in the future if I submit more. Thank you for your comment, though.
              $endgroup$
              – John S.
              2 days ago













            7












            7








            7





            $begingroup$

            The odd one out is the




            green block with 5246.




            This is because




            for all the other numbers on the other blocks, combining the first and last digits and dividing by 2 gave the middle two digits. However, 5246 does not apply to that rule. To conform it to that rule, it would have to be 4248 (one of many possible fixes)...




            Safe Cracked!






            share|improve this answer











            $endgroup$



            The odd one out is the




            green block with 5246.




            This is because




            for all the other numbers on the other blocks, combining the first and last digits and dividing by 2 gave the middle two digits. However, 5246 does not apply to that rule. To conform it to that rule, it would have to be 4248 (one of many possible fixes)...




            Safe Cracked!







            share|improve this answer














            share|improve this answer



            share|improve this answer








            edited May 2 at 23:24

























            answered May 2 at 16:47









            Invent PaloozaInvent Palooza

            21912




            21912











            • $begingroup$
              The answer should be a single block (the ones that have a letter and number); not a complete group of blocks
              $endgroup$
              – Daniel Duque
              May 2 at 17:52










            • $begingroup$
              Yes. Ok, I am giving a very small , yet critical Hint: It's all a numbers game, it's allll a numbers game.
              $endgroup$
              – John S.
              May 2 at 18:32






            • 1




              $begingroup$
              @JohnS. Didn't Ak19 correctly identify this block in an earlier answer?
              $endgroup$
              – noedne
              May 2 at 23:40






            • 2




              $begingroup$
              @JohnS. While Invent's original answer (divisible by 3) is an accurate pattern and a good find, Ak19 posted it first. It's also not the same as your "correct" answer. At all. You just got tired of waiting for someone to discover your solution and decided to give it away to someone.
              $endgroup$
              – David K
              2 days ago






            • 1




              $begingroup$
              No, there are many correct answers, as I have been pointed out. I wlll never make a puzzle with multiple solutions again. Not intentional anyways. He got a correct answer, we can't take that away from him. I am new here, so this is a learning lesson. I will be more specific in the future if I submit more. Thank you for your comment, though.
              $endgroup$
              – John S.
              2 days ago
















            • $begingroup$
              The answer should be a single block (the ones that have a letter and number); not a complete group of blocks
              $endgroup$
              – Daniel Duque
              May 2 at 17:52










            • $begingroup$
              Yes. Ok, I am giving a very small , yet critical Hint: It's all a numbers game, it's allll a numbers game.
              $endgroup$
              – John S.
              May 2 at 18:32






            • 1




              $begingroup$
              @JohnS. Didn't Ak19 correctly identify this block in an earlier answer?
              $endgroup$
              – noedne
              May 2 at 23:40






            • 2




              $begingroup$
              @JohnS. While Invent's original answer (divisible by 3) is an accurate pattern and a good find, Ak19 posted it first. It's also not the same as your "correct" answer. At all. You just got tired of waiting for someone to discover your solution and decided to give it away to someone.
              $endgroup$
              – David K
              2 days ago






            • 1




              $begingroup$
              No, there are many correct answers, as I have been pointed out. I wlll never make a puzzle with multiple solutions again. Not intentional anyways. He got a correct answer, we can't take that away from him. I am new here, so this is a learning lesson. I will be more specific in the future if I submit more. Thank you for your comment, though.
              $endgroup$
              – John S.
              2 days ago















            $begingroup$
            The answer should be a single block (the ones that have a letter and number); not a complete group of blocks
            $endgroup$
            – Daniel Duque
            May 2 at 17:52




            $begingroup$
            The answer should be a single block (the ones that have a letter and number); not a complete group of blocks
            $endgroup$
            – Daniel Duque
            May 2 at 17:52












            $begingroup$
            Yes. Ok, I am giving a very small , yet critical Hint: It's all a numbers game, it's allll a numbers game.
            $endgroup$
            – John S.
            May 2 at 18:32




            $begingroup$
            Yes. Ok, I am giving a very small , yet critical Hint: It's all a numbers game, it's allll a numbers game.
            $endgroup$
            – John S.
            May 2 at 18:32




            1




            1




            $begingroup$
            @JohnS. Didn't Ak19 correctly identify this block in an earlier answer?
            $endgroup$
            – noedne
            May 2 at 23:40




            $begingroup$
            @JohnS. Didn't Ak19 correctly identify this block in an earlier answer?
            $endgroup$
            – noedne
            May 2 at 23:40




            2




            2




            $begingroup$
            @JohnS. While Invent's original answer (divisible by 3) is an accurate pattern and a good find, Ak19 posted it first. It's also not the same as your "correct" answer. At all. You just got tired of waiting for someone to discover your solution and decided to give it away to someone.
            $endgroup$
            – David K
            2 days ago




            $begingroup$
            @JohnS. While Invent's original answer (divisible by 3) is an accurate pattern and a good find, Ak19 posted it first. It's also not the same as your "correct" answer. At all. You just got tired of waiting for someone to discover your solution and decided to give it away to someone.
            $endgroup$
            – David K
            2 days ago




            1




            1




            $begingroup$
            No, there are many correct answers, as I have been pointed out. I wlll never make a puzzle with multiple solutions again. Not intentional anyways. He got a correct answer, we can't take that away from him. I am new here, so this is a learning lesson. I will be more specific in the future if I submit more. Thank you for your comment, though.
            $endgroup$
            – John S.
            2 days ago




            $begingroup$
            No, there are many correct answers, as I have been pointed out. I wlll never make a puzzle with multiple solutions again. Not intentional anyways. He got a correct answer, we can't take that away from him. I am new here, so this is a learning lesson. I will be more specific in the future if I submit more. Thank you for your comment, though.
            $endgroup$
            – John S.
            2 days ago











            9












            $begingroup$

            The




            green block with the number 5246




            doesn't belong, because




            it is the only number not divisible by 3 where all the others are.







            share|improve this answer










            New contributor




            Ak19 is a new contributor to this site. Take care in asking for clarification, commenting, and answering.
            Check out our Code of Conduct.






            $endgroup$












            • $begingroup$
              Ak19, that is a very good suggested answer, however, it is not correct. You are thinking along the correct path, however.
              $endgroup$
              – John S.
              May 2 at 16:48










            • $begingroup$
              @Invent: 99.99999999%
              $endgroup$
              – John S.
              May 2 at 17:55






            • 1




              $begingroup$
              This is the right answer, for a slightly weaker reason than the ‘‘official’’ one.
              $endgroup$
              – Peregrine Rook
              May 3 at 2:20










            • $begingroup$
              all the numbers had exactly 3 prime factors. they were all (2,3,X) for mostly very large X (ranging from 11 to 1583). The only outlier is 5246 with prime factors of (2,43,61) once I saw this I couldn't look away!
              $endgroup$
              – stew
              2 days ago










            • $begingroup$
              @stew: Your comment is essentially the same as David K’s answer.
              $endgroup$
              – Peregrine Rook
              yesterday















            9












            $begingroup$

            The




            green block with the number 5246




            doesn't belong, because




            it is the only number not divisible by 3 where all the others are.







            share|improve this answer










            New contributor




            Ak19 is a new contributor to this site. Take care in asking for clarification, commenting, and answering.
            Check out our Code of Conduct.






            $endgroup$












            • $begingroup$
              Ak19, that is a very good suggested answer, however, it is not correct. You are thinking along the correct path, however.
              $endgroup$
              – John S.
              May 2 at 16:48










            • $begingroup$
              @Invent: 99.99999999%
              $endgroup$
              – John S.
              May 2 at 17:55






            • 1




              $begingroup$
              This is the right answer, for a slightly weaker reason than the ‘‘official’’ one.
              $endgroup$
              – Peregrine Rook
              May 3 at 2:20










            • $begingroup$
              all the numbers had exactly 3 prime factors. they were all (2,3,X) for mostly very large X (ranging from 11 to 1583). The only outlier is 5246 with prime factors of (2,43,61) once I saw this I couldn't look away!
              $endgroup$
              – stew
              2 days ago










            • $begingroup$
              @stew: Your comment is essentially the same as David K’s answer.
              $endgroup$
              – Peregrine Rook
              yesterday













            9












            9








            9





            $begingroup$

            The




            green block with the number 5246




            doesn't belong, because




            it is the only number not divisible by 3 where all the others are.







            share|improve this answer










            New contributor




            Ak19 is a new contributor to this site. Take care in asking for clarification, commenting, and answering.
            Check out our Code of Conduct.






            $endgroup$



            The




            green block with the number 5246




            doesn't belong, because




            it is the only number not divisible by 3 where all the others are.








            share|improve this answer










            New contributor




            Ak19 is a new contributor to this site. Take care in asking for clarification, commenting, and answering.
            Check out our Code of Conduct.









            share|improve this answer



            share|improve this answer








            edited May 2 at 16:44









            Invent Palooza

            21912




            21912






            New contributor




            Ak19 is a new contributor to this site. Take care in asking for clarification, commenting, and answering.
            Check out our Code of Conduct.









            answered May 2 at 16:41









            Ak19Ak19

            2689




            2689




            New contributor




            Ak19 is a new contributor to this site. Take care in asking for clarification, commenting, and answering.
            Check out our Code of Conduct.





            New contributor





            Ak19 is a new contributor to this site. Take care in asking for clarification, commenting, and answering.
            Check out our Code of Conduct.






            Ak19 is a new contributor to this site. Take care in asking for clarification, commenting, and answering.
            Check out our Code of Conduct.











            • $begingroup$
              Ak19, that is a very good suggested answer, however, it is not correct. You are thinking along the correct path, however.
              $endgroup$
              – John S.
              May 2 at 16:48










            • $begingroup$
              @Invent: 99.99999999%
              $endgroup$
              – John S.
              May 2 at 17:55






            • 1




              $begingroup$
              This is the right answer, for a slightly weaker reason than the ‘‘official’’ one.
              $endgroup$
              – Peregrine Rook
              May 3 at 2:20










            • $begingroup$
              all the numbers had exactly 3 prime factors. they were all (2,3,X) for mostly very large X (ranging from 11 to 1583). The only outlier is 5246 with prime factors of (2,43,61) once I saw this I couldn't look away!
              $endgroup$
              – stew
              2 days ago










            • $begingroup$
              @stew: Your comment is essentially the same as David K’s answer.
              $endgroup$
              – Peregrine Rook
              yesterday
















            • $begingroup$
              Ak19, that is a very good suggested answer, however, it is not correct. You are thinking along the correct path, however.
              $endgroup$
              – John S.
              May 2 at 16:48










            • $begingroup$
              @Invent: 99.99999999%
              $endgroup$
              – John S.
              May 2 at 17:55






            • 1




              $begingroup$
              This is the right answer, for a slightly weaker reason than the ‘‘official’’ one.
              $endgroup$
              – Peregrine Rook
              May 3 at 2:20










            • $begingroup$
              all the numbers had exactly 3 prime factors. they were all (2,3,X) for mostly very large X (ranging from 11 to 1583). The only outlier is 5246 with prime factors of (2,43,61) once I saw this I couldn't look away!
              $endgroup$
              – stew
              2 days ago










            • $begingroup$
              @stew: Your comment is essentially the same as David K’s answer.
              $endgroup$
              – Peregrine Rook
              yesterday















            $begingroup$
            Ak19, that is a very good suggested answer, however, it is not correct. You are thinking along the correct path, however.
            $endgroup$
            – John S.
            May 2 at 16:48




            $begingroup$
            Ak19, that is a very good suggested answer, however, it is not correct. You are thinking along the correct path, however.
            $endgroup$
            – John S.
            May 2 at 16:48












            $begingroup$
            @Invent: 99.99999999%
            $endgroup$
            – John S.
            May 2 at 17:55




            $begingroup$
            @Invent: 99.99999999%
            $endgroup$
            – John S.
            May 2 at 17:55




            1




            1




            $begingroup$
            This is the right answer, for a slightly weaker reason than the ‘‘official’’ one.
            $endgroup$
            – Peregrine Rook
            May 3 at 2:20




            $begingroup$
            This is the right answer, for a slightly weaker reason than the ‘‘official’’ one.
            $endgroup$
            – Peregrine Rook
            May 3 at 2:20












            $begingroup$
            all the numbers had exactly 3 prime factors. they were all (2,3,X) for mostly very large X (ranging from 11 to 1583). The only outlier is 5246 with prime factors of (2,43,61) once I saw this I couldn't look away!
            $endgroup$
            – stew
            2 days ago




            $begingroup$
            all the numbers had exactly 3 prime factors. they were all (2,3,X) for mostly very large X (ranging from 11 to 1583). The only outlier is 5246 with prime factors of (2,43,61) once I saw this I couldn't look away!
            $endgroup$
            – stew
            2 days ago












            $begingroup$
            @stew: Your comment is essentially the same as David K’s answer.
            $endgroup$
            – Peregrine Rook
            yesterday




            $begingroup$
            @stew: Your comment is essentially the same as David K’s answer.
            $endgroup$
            – Peregrine Rook
            yesterday











            3












            $begingroup$

            I'm going to guess the:




            Black block.

            It's the only one that isn't a tetris piece :P







            share|improve this answer









            $endgroup$

















              3












              $begingroup$

              I'm going to guess the:




              Black block.

              It's the only one that isn't a tetris piece :P







              share|improve this answer









              $endgroup$















                3












                3








                3





                $begingroup$

                I'm going to guess the:




                Black block.

                It's the only one that isn't a tetris piece :P







                share|improve this answer









                $endgroup$



                I'm going to guess the:




                Black block.

                It's the only one that isn't a tetris piece :P








                share|improve this answer












                share|improve this answer



                share|improve this answer










                answered May 2 at 16:09









                SmockSmock

                5988




                5988





















                    3












                    $begingroup$

                    I will guess that it is




                    Green block 5246




                    My reasoning is that




                    All other blocks have numbers that are divisible by 6, and it is the only block that has two prime factor other than 2 and 3. The prime factors of all other numbers are some number of 2s and 3s, and one other larger prime number. 5246 factorizes to 2, 43, and 61.







                    share|improve this answer









                    $endgroup$












                    • $begingroup$
                      This is equivalent to Ak19’s answer, but worded in a more complicated way.
                      $endgroup$
                      – Peregrine Rook
                      May 3 at 2:21










                    • $begingroup$
                      @PeregrineRook It is an expansion of Ak19's answer, but they are not equivalent. The OP indicated that Ak19 was thinking along the correct path, so I dug deeper in that direction.
                      $endgroup$
                      – David K
                      2 days ago






                    • 1




                      $begingroup$
                      Ah, I see.  OK, you’re right.  Your answer encompasses Ak19’s answer, but you found something more. (Now it turns out that Ak19’s answer was very close to what the OP was looking for, and yours was not, but that doesn’t reflect badly on your answer.  It just goes to support the argument that the question is too broad, because it has too many different answers.) I would upvote you, but I already did.
                      $endgroup$
                      – Peregrine Rook
                      yesterday















                    3












                    $begingroup$

                    I will guess that it is




                    Green block 5246




                    My reasoning is that




                    All other blocks have numbers that are divisible by 6, and it is the only block that has two prime factor other than 2 and 3. The prime factors of all other numbers are some number of 2s and 3s, and one other larger prime number. 5246 factorizes to 2, 43, and 61.







                    share|improve this answer









                    $endgroup$












                    • $begingroup$
                      This is equivalent to Ak19’s answer, but worded in a more complicated way.
                      $endgroup$
                      – Peregrine Rook
                      May 3 at 2:21










                    • $begingroup$
                      @PeregrineRook It is an expansion of Ak19's answer, but they are not equivalent. The OP indicated that Ak19 was thinking along the correct path, so I dug deeper in that direction.
                      $endgroup$
                      – David K
                      2 days ago






                    • 1




                      $begingroup$
                      Ah, I see.  OK, you’re right.  Your answer encompasses Ak19’s answer, but you found something more. (Now it turns out that Ak19’s answer was very close to what the OP was looking for, and yours was not, but that doesn’t reflect badly on your answer.  It just goes to support the argument that the question is too broad, because it has too many different answers.) I would upvote you, but I already did.
                      $endgroup$
                      – Peregrine Rook
                      yesterday













                    3












                    3








                    3





                    $begingroup$

                    I will guess that it is




                    Green block 5246




                    My reasoning is that




                    All other blocks have numbers that are divisible by 6, and it is the only block that has two prime factor other than 2 and 3. The prime factors of all other numbers are some number of 2s and 3s, and one other larger prime number. 5246 factorizes to 2, 43, and 61.







                    share|improve this answer









                    $endgroup$



                    I will guess that it is




                    Green block 5246




                    My reasoning is that




                    All other blocks have numbers that are divisible by 6, and it is the only block that has two prime factor other than 2 and 3. The prime factors of all other numbers are some number of 2s and 3s, and one other larger prime number. 5246 factorizes to 2, 43, and 61.








                    share|improve this answer












                    share|improve this answer



                    share|improve this answer










                    answered May 2 at 20:18









                    David KDavid K

                    24519




                    24519











                    • $begingroup$
                      This is equivalent to Ak19’s answer, but worded in a more complicated way.
                      $endgroup$
                      – Peregrine Rook
                      May 3 at 2:21










                    • $begingroup$
                      @PeregrineRook It is an expansion of Ak19's answer, but they are not equivalent. The OP indicated that Ak19 was thinking along the correct path, so I dug deeper in that direction.
                      $endgroup$
                      – David K
                      2 days ago






                    • 1




                      $begingroup$
                      Ah, I see.  OK, you’re right.  Your answer encompasses Ak19’s answer, but you found something more. (Now it turns out that Ak19’s answer was very close to what the OP was looking for, and yours was not, but that doesn’t reflect badly on your answer.  It just goes to support the argument that the question is too broad, because it has too many different answers.) I would upvote you, but I already did.
                      $endgroup$
                      – Peregrine Rook
                      yesterday
















                    • $begingroup$
                      This is equivalent to Ak19’s answer, but worded in a more complicated way.
                      $endgroup$
                      – Peregrine Rook
                      May 3 at 2:21










                    • $begingroup$
                      @PeregrineRook It is an expansion of Ak19's answer, but they are not equivalent. The OP indicated that Ak19 was thinking along the correct path, so I dug deeper in that direction.
                      $endgroup$
                      – David K
                      2 days ago






                    • 1




                      $begingroup$
                      Ah, I see.  OK, you’re right.  Your answer encompasses Ak19’s answer, but you found something more. (Now it turns out that Ak19’s answer was very close to what the OP was looking for, and yours was not, but that doesn’t reflect badly on your answer.  It just goes to support the argument that the question is too broad, because it has too many different answers.) I would upvote you, but I already did.
                      $endgroup$
                      – Peregrine Rook
                      yesterday















                    $begingroup$
                    This is equivalent to Ak19’s answer, but worded in a more complicated way.
                    $endgroup$
                    – Peregrine Rook
                    May 3 at 2:21




                    $begingroup$
                    This is equivalent to Ak19’s answer, but worded in a more complicated way.
                    $endgroup$
                    – Peregrine Rook
                    May 3 at 2:21












                    $begingroup$
                    @PeregrineRook It is an expansion of Ak19's answer, but they are not equivalent. The OP indicated that Ak19 was thinking along the correct path, so I dug deeper in that direction.
                    $endgroup$
                    – David K
                    2 days ago




                    $begingroup$
                    @PeregrineRook It is an expansion of Ak19's answer, but they are not equivalent. The OP indicated that Ak19 was thinking along the correct path, so I dug deeper in that direction.
                    $endgroup$
                    – David K
                    2 days ago




                    1




                    1




                    $begingroup$
                    Ah, I see.  OK, you’re right.  Your answer encompasses Ak19’s answer, but you found something more. (Now it turns out that Ak19’s answer was very close to what the OP was looking for, and yours was not, but that doesn’t reflect badly on your answer.  It just goes to support the argument that the question is too broad, because it has too many different answers.) I would upvote you, but I already did.
                    $endgroup$
                    – Peregrine Rook
                    yesterday




                    $begingroup$
                    Ah, I see.  OK, you’re right.  Your answer encompasses Ak19’s answer, but you found something more. (Now it turns out that Ak19’s answer was very close to what the OP was looking for, and yours was not, but that doesn’t reflect badly on your answer.  It just goes to support the argument that the question is too broad, because it has too many different answers.) I would upvote you, but I already did.
                    $endgroup$
                    – Peregrine Rook
                    yesterday











                    3












                    $begingroup$

                    My answer:




                    When arranging the numbers in the order of the word Stack Exchange, and then doing an absolute value of the difference between any number and the following number, I get a pattern that the difference value rises then drops alternatingly, except at the Black A where the value remains the same. (Differences: 4176,8214,8214,3150,5088,1950,5274,2152,4252,0,4224,3126 - rise fall neutral fall rise fall rise fall rise fall rise fall)




                    Below is Meant as comment, but alas



                    @John S posted the following in a comment at https://puzzling.stackexchange.com/a/83526/60039




                    [ C, 8412, E, 174,H, 5246,C, 7398,A, 9498,S, 4236,A, 198,T, 8412,N, 9498,G, 5274,E, 2148 ]




                    It is missing data.




                    The last three are the orange blocks, the second to last three are the black blocks, the third to last three are the green blocks and the first 2 are only half of the yellow blocks, the x and k are missing.




                    However:




                    There are two pairs letters with same numbers, 9498 and 8412.




                    Also:




                    John stated elsewhere its just a numbers game...which makes sense because the letters spell out Stack Exchange







                    share|improve this answer










                    New contributor




                    wolfsshield is a new contributor to this site. Take care in asking for clarification, commenting, and answering.
                    Check out our Code of Conduct.






                    $endgroup$

















                      3












                      $begingroup$

                      My answer:




                      When arranging the numbers in the order of the word Stack Exchange, and then doing an absolute value of the difference between any number and the following number, I get a pattern that the difference value rises then drops alternatingly, except at the Black A where the value remains the same. (Differences: 4176,8214,8214,3150,5088,1950,5274,2152,4252,0,4224,3126 - rise fall neutral fall rise fall rise fall rise fall rise fall)




                      Below is Meant as comment, but alas



                      @John S posted the following in a comment at https://puzzling.stackexchange.com/a/83526/60039




                      [ C, 8412, E, 174,H, 5246,C, 7398,A, 9498,S, 4236,A, 198,T, 8412,N, 9498,G, 5274,E, 2148 ]




                      It is missing data.




                      The last three are the orange blocks, the second to last three are the black blocks, the third to last three are the green blocks and the first 2 are only half of the yellow blocks, the x and k are missing.




                      However:




                      There are two pairs letters with same numbers, 9498 and 8412.




                      Also:




                      John stated elsewhere its just a numbers game...which makes sense because the letters spell out Stack Exchange







                      share|improve this answer










                      New contributor




                      wolfsshield is a new contributor to this site. Take care in asking for clarification, commenting, and answering.
                      Check out our Code of Conduct.






                      $endgroup$















                        3












                        3








                        3





                        $begingroup$

                        My answer:




                        When arranging the numbers in the order of the word Stack Exchange, and then doing an absolute value of the difference between any number and the following number, I get a pattern that the difference value rises then drops alternatingly, except at the Black A where the value remains the same. (Differences: 4176,8214,8214,3150,5088,1950,5274,2152,4252,0,4224,3126 - rise fall neutral fall rise fall rise fall rise fall rise fall)




                        Below is Meant as comment, but alas



                        @John S posted the following in a comment at https://puzzling.stackexchange.com/a/83526/60039




                        [ C, 8412, E, 174,H, 5246,C, 7398,A, 9498,S, 4236,A, 198,T, 8412,N, 9498,G, 5274,E, 2148 ]




                        It is missing data.




                        The last three are the orange blocks, the second to last three are the black blocks, the third to last three are the green blocks and the first 2 are only half of the yellow blocks, the x and k are missing.




                        However:




                        There are two pairs letters with same numbers, 9498 and 8412.




                        Also:




                        John stated elsewhere its just a numbers game...which makes sense because the letters spell out Stack Exchange







                        share|improve this answer










                        New contributor




                        wolfsshield is a new contributor to this site. Take care in asking for clarification, commenting, and answering.
                        Check out our Code of Conduct.






                        $endgroup$



                        My answer:




                        When arranging the numbers in the order of the word Stack Exchange, and then doing an absolute value of the difference between any number and the following number, I get a pattern that the difference value rises then drops alternatingly, except at the Black A where the value remains the same. (Differences: 4176,8214,8214,3150,5088,1950,5274,2152,4252,0,4224,3126 - rise fall neutral fall rise fall rise fall rise fall rise fall)




                        Below is Meant as comment, but alas



                        @John S posted the following in a comment at https://puzzling.stackexchange.com/a/83526/60039




                        [ C, 8412, E, 174,H, 5246,C, 7398,A, 9498,S, 4236,A, 198,T, 8412,N, 9498,G, 5274,E, 2148 ]




                        It is missing data.




                        The last three are the orange blocks, the second to last three are the black blocks, the third to last three are the green blocks and the first 2 are only half of the yellow blocks, the x and k are missing.




                        However:




                        There are two pairs letters with same numbers, 9498 and 8412.




                        Also:




                        John stated elsewhere its just a numbers game...which makes sense because the letters spell out Stack Exchange








                        share|improve this answer










                        New contributor




                        wolfsshield is a new contributor to this site. Take care in asking for clarification, commenting, and answering.
                        Check out our Code of Conduct.









                        share|improve this answer



                        share|improve this answer








                        edited May 2 at 20:39





















                        New contributor




                        wolfsshield is a new contributor to this site. Take care in asking for clarification, commenting, and answering.
                        Check out our Code of Conduct.









                        answered May 2 at 20:16









                        wolfsshieldwolfsshield

                        2186




                        2186




                        New contributor




                        wolfsshield is a new contributor to this site. Take care in asking for clarification, commenting, and answering.
                        Check out our Code of Conduct.





                        New contributor





                        wolfsshield is a new contributor to this site. Take care in asking for clarification, commenting, and answering.
                        Check out our Code of Conduct.






                        wolfsshield is a new contributor to this site. Take care in asking for clarification, commenting, and answering.
                        Check out our Code of Conduct.





















                            2












                            $begingroup$

                            The answer is:




                            The green block with the number 9498.




                            because




                            The average of the numbers on the green blocks is not a whole, even number (as it is for the other sets of blocks) and the number 9498 here is a duplicate of another block. This block could be 9496 which would make it valid.







                            share|improve this answer










                            New contributor




                            Amru E. is a new contributor to this site. Take care in asking for clarification, commenting, and answering.
                            Check out our Code of Conduct.






                            $endgroup$

















                              2












                              $begingroup$

                              The answer is:




                              The green block with the number 9498.




                              because




                              The average of the numbers on the green blocks is not a whole, even number (as it is for the other sets of blocks) and the number 9498 here is a duplicate of another block. This block could be 9496 which would make it valid.







                              share|improve this answer










                              New contributor




                              Amru E. is a new contributor to this site. Take care in asking for clarification, commenting, and answering.
                              Check out our Code of Conduct.






                              $endgroup$















                                2












                                2








                                2





                                $begingroup$

                                The answer is:




                                The green block with the number 9498.




                                because




                                The average of the numbers on the green blocks is not a whole, even number (as it is for the other sets of blocks) and the number 9498 here is a duplicate of another block. This block could be 9496 which would make it valid.







                                share|improve this answer










                                New contributor




                                Amru E. is a new contributor to this site. Take care in asking for clarification, commenting, and answering.
                                Check out our Code of Conduct.






                                $endgroup$



                                The answer is:




                                The green block with the number 9498.




                                because




                                The average of the numbers on the green blocks is not a whole, even number (as it is for the other sets of blocks) and the number 9498 here is a duplicate of another block. This block could be 9496 which would make it valid.








                                share|improve this answer










                                New contributor




                                Amru E. is a new contributor to this site. Take care in asking for clarification, commenting, and answering.
                                Check out our Code of Conduct.









                                share|improve this answer



                                share|improve this answer








                                edited May 2 at 20:36





















                                New contributor




                                Amru E. is a new contributor to this site. Take care in asking for clarification, commenting, and answering.
                                Check out our Code of Conduct.









                                answered May 2 at 20:30









                                Amru E.Amru E.

                                1213




                                1213




                                New contributor




                                Amru E. is a new contributor to this site. Take care in asking for clarification, commenting, and answering.
                                Check out our Code of Conduct.





                                New contributor





                                Amru E. is a new contributor to this site. Take care in asking for clarification, commenting, and answering.
                                Check out our Code of Conduct.






                                Amru E. is a new contributor to this site. Take care in asking for clarification, commenting, and answering.
                                Check out our Code of Conduct.





















                                    2












                                    $begingroup$

                                    Answer:




                                    The Black A




                                    Reason:




                                    It's the only one that has 11 as a factor







                                    share|improve this answer








                                    New contributor




                                    wolfsshield is a new contributor to this site. Take care in asking for clarification, commenting, and answering.
                                    Check out our Code of Conduct.






                                    $endgroup$

















                                      2












                                      $begingroup$

                                      Answer:




                                      The Black A




                                      Reason:




                                      It's the only one that has 11 as a factor







                                      share|improve this answer








                                      New contributor




                                      wolfsshield is a new contributor to this site. Take care in asking for clarification, commenting, and answering.
                                      Check out our Code of Conduct.






                                      $endgroup$















                                        2












                                        2








                                        2





                                        $begingroup$

                                        Answer:




                                        The Black A




                                        Reason:




                                        It's the only one that has 11 as a factor







                                        share|improve this answer








                                        New contributor




                                        wolfsshield is a new contributor to this site. Take care in asking for clarification, commenting, and answering.
                                        Check out our Code of Conduct.






                                        $endgroup$



                                        Answer:




                                        The Black A




                                        Reason:




                                        It's the only one that has 11 as a factor








                                        share|improve this answer








                                        New contributor




                                        wolfsshield is a new contributor to this site. Take care in asking for clarification, commenting, and answering.
                                        Check out our Code of Conduct.









                                        share|improve this answer



                                        share|improve this answer






                                        New contributor




                                        wolfsshield is a new contributor to this site. Take care in asking for clarification, commenting, and answering.
                                        Check out our Code of Conduct.









                                        answered May 2 at 21:20









                                        wolfsshieldwolfsshield

                                        2186




                                        2186




                                        New contributor




                                        wolfsshield is a new contributor to this site. Take care in asking for clarification, commenting, and answering.
                                        Check out our Code of Conduct.





                                        New contributor





                                        wolfsshield is a new contributor to this site. Take care in asking for clarification, commenting, and answering.
                                        Check out our Code of Conduct.






                                        wolfsshield is a new contributor to this site. Take care in asking for clarification, commenting, and answering.
                                        Check out our Code of Conduct.





















                                            1












                                            $begingroup$

                                            I would guess it is the:




                                            Yellow block that has the E and the number 174




                                            Why?




                                            All other group of blocks follow the pattern of increasing/decreasing the number as the letters advance through the alphabet.







                                            share|improve this answer









                                            $endgroup$

















                                              1












                                              $begingroup$

                                              I would guess it is the:




                                              Yellow block that has the E and the number 174




                                              Why?




                                              All other group of blocks follow the pattern of increasing/decreasing the number as the letters advance through the alphabet.







                                              share|improve this answer









                                              $endgroup$















                                                1












                                                1








                                                1





                                                $begingroup$

                                                I would guess it is the:




                                                Yellow block that has the E and the number 174




                                                Why?




                                                All other group of blocks follow the pattern of increasing/decreasing the number as the letters advance through the alphabet.







                                                share|improve this answer









                                                $endgroup$



                                                I would guess it is the:




                                                Yellow block that has the E and the number 174




                                                Why?




                                                All other group of blocks follow the pattern of increasing/decreasing the number as the letters advance through the alphabet.








                                                share|improve this answer












                                                share|improve this answer



                                                share|improve this answer










                                                answered May 2 at 17:47









                                                Daniel DuqueDaniel Duque

                                                1565




                                                1565





















                                                    0












                                                    $begingroup$

                                                    The answer might be:




                                                    The yellow one




                                                    Reasoning:




                                                    All the other blocks have only letters when looked at from a specific direction.







                                                    share|improve this answer









                                                    $endgroup$












                                                    • $begingroup$
                                                      No, sorry. We are looking for an individual block that just doesn't fit the criteria of all the others. - But that was a reasonable guess!
                                                      $endgroup$
                                                      – John S.
                                                      May 2 at 15:40











                                                    • $begingroup$
                                                      HERE IS THE DATA SET FOR THIS PUZZLE... Now, we are back to our regularly scheduled "What doesn't belong in this pattern?" puzzle. [ C, 8412, E, 174,H, 5246,C, 7398,A, 9498,S, 4236,A, 198,T, 8412,N, 9498,G, 5274,E, 2148 ]
                                                      $endgroup$
                                                      – John S.
                                                      May 2 at 19:06











                                                    • $begingroup$
                                                      Delete this question. Not editing it. ANSWER WAS SIMPLE: take first number and last number , combine. Divide by 2. will give you middle number. ONLY ONE block doesn't fit criteria. People had fun trying, you can delete. Thanks.
                                                      $endgroup$
                                                      – John S.
                                                      2 days ago















                                                    0












                                                    $begingroup$

                                                    The answer might be:




                                                    The yellow one




                                                    Reasoning:




                                                    All the other blocks have only letters when looked at from a specific direction.







                                                    share|improve this answer









                                                    $endgroup$












                                                    • $begingroup$
                                                      No, sorry. We are looking for an individual block that just doesn't fit the criteria of all the others. - But that was a reasonable guess!
                                                      $endgroup$
                                                      – John S.
                                                      May 2 at 15:40











                                                    • $begingroup$
                                                      HERE IS THE DATA SET FOR THIS PUZZLE... Now, we are back to our regularly scheduled "What doesn't belong in this pattern?" puzzle. [ C, 8412, E, 174,H, 5246,C, 7398,A, 9498,S, 4236,A, 198,T, 8412,N, 9498,G, 5274,E, 2148 ]
                                                      $endgroup$
                                                      – John S.
                                                      May 2 at 19:06











                                                    • $begingroup$
                                                      Delete this question. Not editing it. ANSWER WAS SIMPLE: take first number and last number , combine. Divide by 2. will give you middle number. ONLY ONE block doesn't fit criteria. People had fun trying, you can delete. Thanks.
                                                      $endgroup$
                                                      – John S.
                                                      2 days ago













                                                    0












                                                    0








                                                    0





                                                    $begingroup$

                                                    The answer might be:




                                                    The yellow one




                                                    Reasoning:




                                                    All the other blocks have only letters when looked at from a specific direction.







                                                    share|improve this answer









                                                    $endgroup$



                                                    The answer might be:




                                                    The yellow one




                                                    Reasoning:




                                                    All the other blocks have only letters when looked at from a specific direction.








                                                    share|improve this answer












                                                    share|improve this answer



                                                    share|improve this answer










                                                    answered May 2 at 15:39









                                                    Krad CigolKrad Cigol

                                                    1,096212




                                                    1,096212











                                                    • $begingroup$
                                                      No, sorry. We are looking for an individual block that just doesn't fit the criteria of all the others. - But that was a reasonable guess!
                                                      $endgroup$
                                                      – John S.
                                                      May 2 at 15:40











                                                    • $begingroup$
                                                      HERE IS THE DATA SET FOR THIS PUZZLE... Now, we are back to our regularly scheduled "What doesn't belong in this pattern?" puzzle. [ C, 8412, E, 174,H, 5246,C, 7398,A, 9498,S, 4236,A, 198,T, 8412,N, 9498,G, 5274,E, 2148 ]
                                                      $endgroup$
                                                      – John S.
                                                      May 2 at 19:06











                                                    • $begingroup$
                                                      Delete this question. Not editing it. ANSWER WAS SIMPLE: take first number and last number , combine. Divide by 2. will give you middle number. ONLY ONE block doesn't fit criteria. People had fun trying, you can delete. Thanks.
                                                      $endgroup$
                                                      – John S.
                                                      2 days ago
















                                                    • $begingroup$
                                                      No, sorry. We are looking for an individual block that just doesn't fit the criteria of all the others. - But that was a reasonable guess!
                                                      $endgroup$
                                                      – John S.
                                                      May 2 at 15:40











                                                    • $begingroup$
                                                      HERE IS THE DATA SET FOR THIS PUZZLE... Now, we are back to our regularly scheduled "What doesn't belong in this pattern?" puzzle. [ C, 8412, E, 174,H, 5246,C, 7398,A, 9498,S, 4236,A, 198,T, 8412,N, 9498,G, 5274,E, 2148 ]
                                                      $endgroup$
                                                      – John S.
                                                      May 2 at 19:06











                                                    • $begingroup$
                                                      Delete this question. Not editing it. ANSWER WAS SIMPLE: take first number and last number , combine. Divide by 2. will give you middle number. ONLY ONE block doesn't fit criteria. People had fun trying, you can delete. Thanks.
                                                      $endgroup$
                                                      – John S.
                                                      2 days ago















                                                    $begingroup$
                                                    No, sorry. We are looking for an individual block that just doesn't fit the criteria of all the others. - But that was a reasonable guess!
                                                    $endgroup$
                                                    – John S.
                                                    May 2 at 15:40





                                                    $begingroup$
                                                    No, sorry. We are looking for an individual block that just doesn't fit the criteria of all the others. - But that was a reasonable guess!
                                                    $endgroup$
                                                    – John S.
                                                    May 2 at 15:40













                                                    $begingroup$
                                                    HERE IS THE DATA SET FOR THIS PUZZLE... Now, we are back to our regularly scheduled "What doesn't belong in this pattern?" puzzle. [ C, 8412, E, 174,H, 5246,C, 7398,A, 9498,S, 4236,A, 198,T, 8412,N, 9498,G, 5274,E, 2148 ]
                                                    $endgroup$
                                                    – John S.
                                                    May 2 at 19:06





                                                    $begingroup$
                                                    HERE IS THE DATA SET FOR THIS PUZZLE... Now, we are back to our regularly scheduled "What doesn't belong in this pattern?" puzzle. [ C, 8412, E, 174,H, 5246,C, 7398,A, 9498,S, 4236,A, 198,T, 8412,N, 9498,G, 5274,E, 2148 ]
                                                    $endgroup$
                                                    – John S.
                                                    May 2 at 19:06













                                                    $begingroup$
                                                    Delete this question. Not editing it. ANSWER WAS SIMPLE: take first number and last number , combine. Divide by 2. will give you middle number. ONLY ONE block doesn't fit criteria. People had fun trying, you can delete. Thanks.
                                                    $endgroup$
                                                    – John S.
                                                    2 days ago




                                                    $begingroup$
                                                    Delete this question. Not editing it. ANSWER WAS SIMPLE: take first number and last number , combine. Divide by 2. will give you middle number. ONLY ONE block doesn't fit criteria. People had fun trying, you can delete. Thanks.
                                                    $endgroup$
                                                    – John S.
                                                    2 days ago











                                                    0












                                                    $begingroup$

                                                    I noticed that




                                                    if you open out the net of each, centred on the side showing the letter, any numbers above, below and to the right are all aligned in the same direction as the letter; and the number on the left is on its side, with its top pointing to the left. The only exception is the Yellow K block, where the number on the right isn't aligned in the same direction as the letter.







                                                    share|improve this answer








                                                    New contributor




                                                    MichaelMaggs is a new contributor to this site. Take care in asking for clarification, commenting, and answering.
                                                    Check out our Code of Conduct.






                                                    $endgroup$








                                                    • 1




                                                      $begingroup$
                                                      Looks like there is more than one valid solution :(
                                                      $endgroup$
                                                      – MichaelMaggs
                                                      May 2 at 16:51






                                                    • 5




                                                      $begingroup$
                                                      If there's more than one block that's different from the others in a notable way and the challenge is to figure out what specific difference you had in mind, then the puzzle is basically "read the puzzle author's mind". Let's hope it turns out that the odd-one-out property John S intends is much more compelling than the two other perfectly plausible ones suggested so far...
                                                      $endgroup$
                                                      – Gareth McCaughan
                                                      May 2 at 17:42






                                                    • 1




                                                      $begingroup$
                                                      It is the other way around >! This is a hint.
                                                      $endgroup$
                                                      – Daniel Duque
                                                      May 2 at 18:33






                                                    • 1




                                                      $begingroup$
                                                      @JohnS. typically we use Rot13 to hide hints or spoilers in comments. But I’d strongly recommend you put the hints in the original post, since they would be considered part of the overall puzzle
                                                      $endgroup$
                                                      – PiIsNot3
                                                      May 2 at 19:12







                                                    • 1




                                                      $begingroup$
                                                      @JohnS. A "What's next?" like you describe should also be closed for being too broad...
                                                      $endgroup$
                                                      – noedne
                                                      May 2 at 21:24















                                                    0












                                                    $begingroup$

                                                    I noticed that




                                                    if you open out the net of each, centred on the side showing the letter, any numbers above, below and to the right are all aligned in the same direction as the letter; and the number on the left is on its side, with its top pointing to the left. The only exception is the Yellow K block, where the number on the right isn't aligned in the same direction as the letter.







                                                    share|improve this answer








                                                    New contributor




                                                    MichaelMaggs is a new contributor to this site. Take care in asking for clarification, commenting, and answering.
                                                    Check out our Code of Conduct.






                                                    $endgroup$








                                                    • 1




                                                      $begingroup$
                                                      Looks like there is more than one valid solution :(
                                                      $endgroup$
                                                      – MichaelMaggs
                                                      May 2 at 16:51






                                                    • 5




                                                      $begingroup$
                                                      If there's more than one block that's different from the others in a notable way and the challenge is to figure out what specific difference you had in mind, then the puzzle is basically "read the puzzle author's mind". Let's hope it turns out that the odd-one-out property John S intends is much more compelling than the two other perfectly plausible ones suggested so far...
                                                      $endgroup$
                                                      – Gareth McCaughan
                                                      May 2 at 17:42






                                                    • 1




                                                      $begingroup$
                                                      It is the other way around >! This is a hint.
                                                      $endgroup$
                                                      – Daniel Duque
                                                      May 2 at 18:33






                                                    • 1




                                                      $begingroup$
                                                      @JohnS. typically we use Rot13 to hide hints or spoilers in comments. But I’d strongly recommend you put the hints in the original post, since they would be considered part of the overall puzzle
                                                      $endgroup$
                                                      – PiIsNot3
                                                      May 2 at 19:12







                                                    • 1




                                                      $begingroup$
                                                      @JohnS. A "What's next?" like you describe should also be closed for being too broad...
                                                      $endgroup$
                                                      – noedne
                                                      May 2 at 21:24













                                                    0












                                                    0








                                                    0





                                                    $begingroup$

                                                    I noticed that




                                                    if you open out the net of each, centred on the side showing the letter, any numbers above, below and to the right are all aligned in the same direction as the letter; and the number on the left is on its side, with its top pointing to the left. The only exception is the Yellow K block, where the number on the right isn't aligned in the same direction as the letter.







                                                    share|improve this answer








                                                    New contributor




                                                    MichaelMaggs is a new contributor to this site. Take care in asking for clarification, commenting, and answering.
                                                    Check out our Code of Conduct.






                                                    $endgroup$



                                                    I noticed that




                                                    if you open out the net of each, centred on the side showing the letter, any numbers above, below and to the right are all aligned in the same direction as the letter; and the number on the left is on its side, with its top pointing to the left. The only exception is the Yellow K block, where the number on the right isn't aligned in the same direction as the letter.








                                                    share|improve this answer








                                                    New contributor




                                                    MichaelMaggs is a new contributor to this site. Take care in asking for clarification, commenting, and answering.
                                                    Check out our Code of Conduct.









                                                    share|improve this answer



                                                    share|improve this answer






                                                    New contributor




                                                    MichaelMaggs is a new contributor to this site. Take care in asking for clarification, commenting, and answering.
                                                    Check out our Code of Conduct.









                                                    answered May 2 at 16:39









                                                    MichaelMaggsMichaelMaggs

                                                    19311




                                                    19311




                                                    New contributor




                                                    MichaelMaggs is a new contributor to this site. Take care in asking for clarification, commenting, and answering.
                                                    Check out our Code of Conduct.





                                                    New contributor





                                                    MichaelMaggs is a new contributor to this site. Take care in asking for clarification, commenting, and answering.
                                                    Check out our Code of Conduct.






                                                    MichaelMaggs is a new contributor to this site. Take care in asking for clarification, commenting, and answering.
                                                    Check out our Code of Conduct.







                                                    • 1




                                                      $begingroup$
                                                      Looks like there is more than one valid solution :(
                                                      $endgroup$
                                                      – MichaelMaggs
                                                      May 2 at 16:51






                                                    • 5




                                                      $begingroup$
                                                      If there's more than one block that's different from the others in a notable way and the challenge is to figure out what specific difference you had in mind, then the puzzle is basically "read the puzzle author's mind". Let's hope it turns out that the odd-one-out property John S intends is much more compelling than the two other perfectly plausible ones suggested so far...
                                                      $endgroup$
                                                      – Gareth McCaughan
                                                      May 2 at 17:42






                                                    • 1




                                                      $begingroup$
                                                      It is the other way around >! This is a hint.
                                                      $endgroup$
                                                      – Daniel Duque
                                                      May 2 at 18:33






                                                    • 1




                                                      $begingroup$
                                                      @JohnS. typically we use Rot13 to hide hints or spoilers in comments. But I’d strongly recommend you put the hints in the original post, since they would be considered part of the overall puzzle
                                                      $endgroup$
                                                      – PiIsNot3
                                                      May 2 at 19:12







                                                    • 1




                                                      $begingroup$
                                                      @JohnS. A "What's next?" like you describe should also be closed for being too broad...
                                                      $endgroup$
                                                      – noedne
                                                      May 2 at 21:24












                                                    • 1




                                                      $begingroup$
                                                      Looks like there is more than one valid solution :(
                                                      $endgroup$
                                                      – MichaelMaggs
                                                      May 2 at 16:51






                                                    • 5




                                                      $begingroup$
                                                      If there's more than one block that's different from the others in a notable way and the challenge is to figure out what specific difference you had in mind, then the puzzle is basically "read the puzzle author's mind". Let's hope it turns out that the odd-one-out property John S intends is much more compelling than the two other perfectly plausible ones suggested so far...
                                                      $endgroup$
                                                      – Gareth McCaughan
                                                      May 2 at 17:42






                                                    • 1




                                                      $begingroup$
                                                      It is the other way around >! This is a hint.
                                                      $endgroup$
                                                      – Daniel Duque
                                                      May 2 at 18:33






                                                    • 1




                                                      $begingroup$
                                                      @JohnS. typically we use Rot13 to hide hints or spoilers in comments. But I’d strongly recommend you put the hints in the original post, since they would be considered part of the overall puzzle
                                                      $endgroup$
                                                      – PiIsNot3
                                                      May 2 at 19:12







                                                    • 1




                                                      $begingroup$
                                                      @JohnS. A "What's next?" like you describe should also be closed for being too broad...
                                                      $endgroup$
                                                      – noedne
                                                      May 2 at 21:24







                                                    1




                                                    1




                                                    $begingroup$
                                                    Looks like there is more than one valid solution :(
                                                    $endgroup$
                                                    – MichaelMaggs
                                                    May 2 at 16:51




                                                    $begingroup$
                                                    Looks like there is more than one valid solution :(
                                                    $endgroup$
                                                    – MichaelMaggs
                                                    May 2 at 16:51




                                                    5




                                                    5




                                                    $begingroup$
                                                    If there's more than one block that's different from the others in a notable way and the challenge is to figure out what specific difference you had in mind, then the puzzle is basically "read the puzzle author's mind". Let's hope it turns out that the odd-one-out property John S intends is much more compelling than the two other perfectly plausible ones suggested so far...
                                                    $endgroup$
                                                    – Gareth McCaughan
                                                    May 2 at 17:42




                                                    $begingroup$
                                                    If there's more than one block that's different from the others in a notable way and the challenge is to figure out what specific difference you had in mind, then the puzzle is basically "read the puzzle author's mind". Let's hope it turns out that the odd-one-out property John S intends is much more compelling than the two other perfectly plausible ones suggested so far...
                                                    $endgroup$
                                                    – Gareth McCaughan
                                                    May 2 at 17:42




                                                    1




                                                    1




                                                    $begingroup$
                                                    It is the other way around >! This is a hint.
                                                    $endgroup$
                                                    – Daniel Duque
                                                    May 2 at 18:33




                                                    $begingroup$
                                                    It is the other way around >! This is a hint.
                                                    $endgroup$
                                                    – Daniel Duque
                                                    May 2 at 18:33




                                                    1




                                                    1




                                                    $begingroup$
                                                    @JohnS. typically we use Rot13 to hide hints or spoilers in comments. But I’d strongly recommend you put the hints in the original post, since they would be considered part of the overall puzzle
                                                    $endgroup$
                                                    – PiIsNot3
                                                    May 2 at 19:12





                                                    $begingroup$
                                                    @JohnS. typically we use Rot13 to hide hints or spoilers in comments. But I’d strongly recommend you put the hints in the original post, since they would be considered part of the overall puzzle
                                                    $endgroup$
                                                    – PiIsNot3
                                                    May 2 at 19:12





                                                    1




                                                    1




                                                    $begingroup$
                                                    @JohnS. A "What's next?" like you describe should also be closed for being too broad...
                                                    $endgroup$
                                                    – noedne
                                                    May 2 at 21:24




                                                    $begingroup$
                                                    @JohnS. A "What's next?" like you describe should also be closed for being too broad...
                                                    $endgroup$
                                                    – noedne
                                                    May 2 at 21:24



                                                    Popular posts from this blog

                                                    Get product attribute by attribute group code in magento 2get product attribute by product attribute group in magento 2Magento 2 Log Bundle Product Data in List Page?How to get all product attribute of a attribute group of Default attribute set?Magento 2.1 Create a filter in the product grid by new attributeMagento 2 : Get Product Attribute values By GroupMagento 2 How to get all existing values for one attributeMagento 2 get custom attribute of a single product inside a pluginMagento 2.3 How to get all the Multi Source Inventory (MSI) locations collection in custom module?Magento2: how to develop rest API to get new productsGet product attribute by attribute group code ( [attribute_group_code] ) in magento 2

                                                    Category:9 (number) SubcategoriesMedia in category "9 (number)"Navigation menuUpload mediaGND ID: 4485639-8Library of Congress authority ID: sh85091979ReasonatorScholiaStatistics

                                                    Magento 2.3: How do i solve this, Not registered handle, on custom form?How can i rewrite TierPrice Block in Magento2magento 2 captcha not rendering if I override layout xmlmain.CRITICAL: Plugin class doesn't existMagento 2 : Problem while adding custom button order view page?Magento 2.2.5: Overriding Admin Controller sales/orderMagento 2.2.5: Add, Update and Delete existing products Custom OptionsMagento 2.3 : File Upload issue in UI Component FormMagento2 Not registered handleHow to configured Form Builder Js in my custom magento 2.3.0 module?Magento 2.3. How to create image upload field in an admin form